Gambia and Juana

This topic has expert replies
Senior | Next Rank: 100 Posts
Posts: 41
Joined: Mon Mar 01, 2010 5:57 pm
Location: US
Thanked: 1 times
GMAT Score:640

Gambia and Juana

by 2011mbaspirant » Thu Jul 28, 2011 10:52 am
Last year, Gambia received $2.5 billion in loans from the International Third World Banking Fund, and its Gross Domestic Product grew by 5%. This year Gambia has requested twice as much money from the ITWBF, and its leaders expect that Gambia's GDP will rise by a full 10%.
Which of the following, if true, would LEAST undermine the expectations of Gambia's leaders?
(A) The large 5% increase of last year is attributable to extraordinary harvests due to unusually good weather conditions.
(B) Gambia's economy is not strong enough to absorb more than $3 billion in outside capital each year.
(C) Gambia does not have sufficient heavy industry to fuel an increase in its GDP of more than 6% per year.
(D) A provision of the charter of the International Third World Banking Fund prohibits the Fund from increasing loans to a country by more than 50% in a single year.
(E) A neighboring country experienced an increase of 5% in its Gross Domestic Product two years ago but an increase of only 3% in the most recent year.
[spoiler] OA is (E), why not (A) ?[/spoiler]

Juana is dining at a Chinese restaurant. She will order either combination platter #2 or combination platter #5, but not both. If she orders combination platter #2, she will eat fried rice. If she orders combination platter #5, she will eat an egg roll.
Given the statements above, which of the following must be true?
(A) Juana will eat either fried rice or an egg roll but not both.
(B) If Juana eats an egg roll, then she ordered combination platter #5.
(C) If Juana does not eat an egg roll, then she ordered combination platter #2.
(D) If Juana eats fried rice, then she ordered combination platter #2.
(E) Anyone who orders combination platter #2 eats fried rice.
[spoiler]OA is (C), why not (A)?[/spoiler]

Master | Next Rank: 500 Posts
Posts: 189
Joined: Wed Jul 06, 2011 6:57 am
Thanked: 17 times
Followed by:1 members

by gmat25 » Thu Jul 28, 2011 11:23 am
Last year, Gambia received $2.5 billion in loans from the International Third World Banking Fund, and its Gross Domestic Product grew by 5%. This year Gambia has requested twice as much money from the ITWBF, and its leaders expect that Gambia's GDP will rise by a full 10%.
Which of the following, if true, would LEAST undermine the expectations of Gambia's leaders?
(A) The large 5% increase of last year is attributable to extraordinary harvests due to unusually good weather conditions.
(B) Gambia's economy is not strong enough to absorb more than $3 billion in outside capital each year.
(C) Gambia does not have sufficient heavy industry to fuel an increase in its GDP of more than 6% per year.
(D) A provision of the charter of the International Third World Banking Fund prohibits the Fund from increasing loans to a country by more than 50% in a single year.
(E) A neighboring country experienced an increase of 5% in its Gross Domestic Product two years ago but an increase of only 3% in the most recent year.
[spoiler] OA is (E), why not (A) ?[/spoiler]
Simple X causes Y situation.

Leaders believe

X (Loan) ----> Y (growth)

hence conclude, 2X ----> 2Y

Op A says, Z causes Y, so Op A is weakening the leaders' conclusion, hence incorrect.
Princeton Review CAT - 710(Q-51, V-37) --> silly mistakes screwed up my VERBAL

User avatar
Master | Next Rank: 500 Posts
Posts: 489
Joined: Tue Jul 05, 2011 11:10 am
Thanked: 28 times
Followed by:5 members

by gmatblood » Thu Jul 28, 2011 12:10 pm
How is E the OA?

Junior | Next Rank: 30 Posts
Posts: 15
Joined: Wed Nov 10, 2010 7:06 am
Thanked: 1 times

by ssidda01 » Thu Jul 28, 2011 12:41 pm
Statement A - 5% increase in GDP is attributed to good harvest in GAMBIA. This, if true, can mean that the loan amount was not necessarily the cause for increase in the GDP.

Statement B - GAMBIA's economy cannot $3 billion captiol each year. This, if true, means that if $2.5 billion can cause 5% increase then to reach 10% increase, GAMBIA would need more than $3 billion.

Statement C - This states that, only heavy industries in GAMBIA can fuel more than 6% increase in GDP per year. This, if true, maximizes the GDP of the country to 6% inspite of the loan amount being twice as much.

Statement D - The ITWBF limits the fund amount to no more than 50% in a single year. This, if true, will not increase the GDP % as the amount of the loan will not go more than $3.75 billion.

Statement E - A neighboring countries GDP increased by only 3% in the most recent years. This, if true, does not affect GAMBIA's chances of increasing its GDP to 10%. This statement, hence, least undermines.

Therefore OA is E.

User avatar
GMAT Instructor
Posts: 1031
Joined: Thu Jul 03, 2008 1:23 pm
Location: Malibu, CA
Thanked: 716 times
Followed by:255 members
GMAT Score:750

by Brian@VeritasPrep » Fri Jul 29, 2011 2:57 pm
Thanks for the PM on #2. #2 has a really strong LSAT feel to it so if it feels a little extra abstract or tricky that's why - I hesitate to say that the GMAT *won't* test this, but it does seem slightly out of the GMAT scope based on what it's tested to date. This is a bit more formal-logic-heavy. But here's the explanation.

We know that:

If platter #2 ---> Fried Rice
If platter #5 ---> Egg Roll

But we don't know the opposites to be true. We don't know that "If Fried Rice --> platter #2". It could be that meal #2 has both fried rice and an egg roll. That wouldn't violate the given information that "if she gets #2 she'll eat fried rice".

As an aside, it could be like saying: "If John orders a cheeseburger, he'll eat cheese. If he orders a hamburger, he'll eat meat." That's true...but not comprehensive. You can't say, "well, then if he ate meat he must have ordered the hamburger." Why not? The cheeseburger has meat too.

So because in these If X ---> Y situations we only know the information in one direction, we don't know A, B, D, or E to be necessarily true.

We do know C, though. Because we know that:

If platter #5 ---> Egg Roll

We can say that the "then" in the "if...then" statement is a necessary condition of the "if". We know that whenever she orders platter 5 she will - therefore "MUST" if you're thinking logically - eat an egg roll. So if she didn't eat the egg roll, that violates a necessary condition of the "Ordered #5" logic. It means that she cannot have ordered #5. Therefore she must have ordered #2.

Logically this is called the "contrapositive". In a certain If X, Then Y statement:

If it's an apple, then it must be a fruit

You know, logically, that:

In order to be an apple, it must be a fruit.

Well, if it must be a fruit in order to be an apple, then if it's not a fruit, we can prove that it's not an apple:

Not fruit ---> Not apple

That's the contrapositive at work. If you know that:

If X, then Y

or All Xs are Ys

Then you can say

In order to X, one must Y

and, therefore, that

If not Y, then not X

So by that logic we can prove C to be true, but we can't prove the others. Like I said, this may be a little more LSAT than GMAT so I wouldn't stress the contrapositive too much, but if you get that logic that's a really good sign...
Last edited by Brian@VeritasPrep on Fri Jul 29, 2011 4:59 pm, edited 1 time in total.
Brian Galvin
GMAT Instructor
Chief Academic Officer
Veritas Prep

Looking for GMAT practice questions? Try out the Veritas Prep Question Bank. Learn More.

Master | Next Rank: 500 Posts
Posts: 189
Joined: Wed Jul 06, 2011 6:57 am
Thanked: 17 times
Followed by:1 members

by gmat25 » Fri Jul 29, 2011 3:23 pm
Hey Brian,

Thanks a lot for that great post. Your post was really helpful. I noted down the takeaway

If X then Y, then logically we can say

Not Y ---> Not X


One more thing, by mistake, i think you made a typo here, if so please edit so that in future many other students who gonna read your explanation and get benefited will not confuse.

If platter #2 ---> Egg Roll

We can say that the "then" in the "if...then" statement is a necessary condition of the "if". We know that whenever she orders platter 2 she will - therefore "MUST" if you're thinking logically - eat an egg roll. So if she didn't eat the egg roll, that violates a necessary condition of the "Ordered #2" logic. It means that she cannot have ordered #2.
It should be If platter #5 ---> Egg Roll


Thanks a lot again!!!
Princeton Review CAT - 710(Q-51, V-37) --> silly mistakes screwed up my VERBAL

User avatar
GMAT Instructor
Posts: 1031
Joined: Thu Jul 03, 2008 1:23 pm
Location: Malibu, CA
Thanked: 716 times
Followed by:255 members
GMAT Score:750

by Brian@VeritasPrep » Fri Jul 29, 2011 5:01 pm
Hey, thanks for catching that, gmat25 - just fixed that. When you're typing in this window sometimes the question scrolls so far up to the top that you try to quote from memory and...well...I learned my lesson! You're right - I meant #5. Fixed!
Brian Galvin
GMAT Instructor
Chief Academic Officer
Veritas Prep

Looking for GMAT practice questions? Try out the Veritas Prep Question Bank. Learn More.

Legendary Member
Posts: 1404
Joined: Tue May 20, 2008 6:55 pm
Thanked: 18 times
Followed by:2 members

by tanviet » Fri Jul 29, 2011 7:18 pm
I do not like both questions

question 1 contain choice D which is contradict evidence and which is never considered a weakener. look at the gmatprep questions for this

question 2 contain formal logic/contrapostive of lsat which is outside scope of gmat.

I think we will waist our precious times for these.

Senior | Next Rank: 100 Posts
Posts: 40
Joined: Thu Jan 21, 2010 1:36 am
Thanked: 6 times
Followed by:1 members

by Prashant Ranjan » Fri Jul 29, 2011 10:47 pm
In my perspective:

Although the two questions here may require little thought provocation but i feel they are not out of scope of GMAT.
Remember to score in 700s one must prepare to reach above 750s. The questions though entail the logic of LSAT but may appear in 700 - 800 question level (may be with a twisted logic rather than employing the blunt concept of Contra-positives)

Thanks

Legendary Member
Posts: 627
Joined: Thu Jun 23, 2011 9:12 am
Thanked: 4 times
Followed by:1 members

by mankey » Wed Feb 01, 2012 9:30 am
Some expert please help with Question 1.

Thanks.

User avatar
Legendary Member
Posts: 934
Joined: Tue Nov 09, 2010 5:16 am
Location: AAMCHI MUMBAI LOCAL
Thanked: 63 times
Followed by:14 members

by [email protected] » Wed Feb 01, 2012 8:37 pm
Last year, Gambia received $2.5 billion in loans from the International Third World Banking Fund, and its Gross Domestic Product grew by 5%. This year Gambia has requested twice as much money from the ITWBF, and its leaders expect that Gambia's GDP will rise by a full 10%.

Which of the following, if true, would LEAST undermine the expectations of Gambia's leaders?

(A) The large 5% increase of last year is attributable to extraordinary harvests due to unusually good weather conditions.

(B) Gambia's economy is not strong enough to absorb more than $3 billion in outside capital each year.

(C) Gambia does not have sufficient heavy industry to fuel an increase in its GDP of more than 6% per year.

(D) A provision of the charter of the International Third World Banking Fund prohibits the Fund from increasing loans to a country by more than 50% in a single year.

(E) A neighboring country experienced an increase of 5% in its Gross Domestic Product two years ago but an increase of only 3% in the most recent year.
OA IS E.



Experts please help as very much confused between options D and E.
Not understanding at all the difference between D and E with reference to the stimulus given
IT IS TIME TO BEAT THE GMAT

LEARNING, APPLICATION AND TIMING IS THE FACT OF GMAT AND LIFE AS WELL... KEEP PLAYING!!!

Whenever you feel that my post really helped you to learn something new, please press on the 'THANK' button.

Legendary Member
Posts: 627
Joined: Thu Jun 23, 2011 9:12 am
Thanked: 4 times
Followed by:1 members

by mankey » Thu Feb 23, 2012 11:10 am
Some expert, please help with Choice E in Question 2.

Regards.

Legendary Member
Posts: 2789
Joined: Tue Jul 26, 2011 12:19 am
Location: Chennai, India
Thanked: 206 times
Followed by:43 members
GMAT Score:640

by GmatKiss » Sat Feb 25, 2012 6:30 am
why C for Problem 2?